You are on page 1of 4

Physics 3323, Fall 2016 Problem Set 8 due Oct 21, 2014

Reading: Griffiths Chapter 5, 6.1-6.2 ~ ·A


a) Does this potential satisfy the gauge choice ∇ ~ = 0?
b) What is the magnetic field?
1. Charged spinning shell
SOLUTION:
Griffiths 5.37b (previously 5.36) Find the magnetic dipole mo-
ment of a spherical shell of radius R spinning with frequency a) We can use the vector product rules found in the front
~
cover of Griffiths to evaluate ∇ · A
ω, with uniform surface charge density σ. Show that for points
r > R the potential is that of a perfect dipole.
 
~ 1 1 1
∇·A=∇· ~c × ~r = ~r(∇ × ~c) − ~c(∇ × ~r) (2.1)
2 2 2
SOLUTION:
Suppose that the sphere is spinning about its z-axis. The ring Now ∇ ×~c = 0 since ~c is a constant vector and you can check
at polar angle θ has charge that ∇ × ~r = 0. Yes, this potential satisfies the gauge choice
~ ·A
∇ ~=0
dq = 2πR sin θ(Rdθ)σ.
b) To get the magnetic field, we just need to take the curl of
~
A.
The current of the ring is
B~ =∇~ ×A ~ = 1∇ ~ × (~c × ~r) (2.2)
ω 2
dI = dq .
2π 1h i
= ~ c − (~c · ∇)~
(~r · ∇)~ ~ r + ~c(∇~ · ~r) − ~r(∇
~ · ~c) (2.3)
The area of the ring is 2
Now the first and last terms in Eq. (2.3) are both 0 since ~c is
A = π(R cos θ)2 a constant vector. The third term is 3~c since ∇ ~ · ~r = 3. The
The magnetic moment of the sphere is second term can be calculated as follows:
 
Z
ω 4πR4 ωσ ~ ∂ ∂ ∂
m = (πR2 sin2 θ)(2πR2 sin θ) σdθ = . (~c · ∇)~r = cx + cy + cz (xx̂ + yŷ + zẑ) (2.4)
2π 3 ∂x ∂y ∂z

= cx x̂ + cy ŷ + cz ẑ = ~c (2.5)
Eq. (2.3) is
2. A vector potential ~ = 1 [−~c + 3~c] = ~c
B (2.6)
~ r) = 1 2
Consider the vector potential A(~ 2
~c × ~r, where ~c is a
constant vector.

1
3. Vector potential of infinite solenoid The solution is:
You have an infinitely long solenoid with radius R and N turns (
µ0 nIs
~= 2 φ̂ inside
of wire per unit length. The wire carries a current I. A µ0 nIR2 (3.5)
a) Find the vector potential of this solenoid. Avoid looking 2s φ̂ outside
~ · d~` =
H
up the answer. It is useful to begin by showing that A
~ · n̂dA, where the LHS integral is around the edge of an b) We didn’t justify why the vector potential should point
R
B
open surface with area A. circumferentially in the last part. The only justification we
b) Compute ∇ ~ ×A ~ in cylindrical coordinates to show that need is to see if taking the curl of the vector potential gives
the correct magnetic field.
you get the correct magnetic field.
~ =∇
B ~ = − ∂Aφ ŝ + 1 ∂ (sAφ ) ẑ
~ ×A (3.6)
SOLUTION: ∂z s ∂s
a) We know the magnetic field in the solenoid (Griffiths Ex- Inside:
µ0 nIs2
 
ample 5.9). ~ =1 ∂

µ0 nI ẑ inside B ẑ = µ0 nI ẑ (3.7)
~
B= (3.1) s ∂s 2
0 outside
Outside:
~ · d~` = B~ · n̂dA as per the hint.
H R
Let’s start by showing A µ0 nIR2
 
~ =1 ∂
B ẑ = 0 (3.8)
This is an application of Stokes theorem to the definition of s ∂s 2
the vector potential
Z Z   I Comparison with Eq. (3.1) shows that this is indeed a correct
~
B · n̂ dA = ~ ~
∇ × A · n̂ dA = A ~ · d~` (3.2) vector potential.

Now the vector potential generally points in the direction of


~ = Aφ φ̂, that is the vector
the current, so let’s suppose that A 4. Square current loop
potential points circumferentially. We create circumferential Griffiths 5.36. Find the exact magnetic field a distance d above
Ampèrian loops. Inside the solenoid Eq. (3.2) becomes the the center of a square loop of side w, carrying a current I. Ver-
following:
ify that it reduces to the field of a dipole with the appropriate
µ0 nIs dipole moment, when z  w.
µ0 nI πs2 = 2πsAφ ⇒ Ap hi = (3.3)
2
Outside the solenoid we get: SOLUTION:
Place the loop in the x-y plane with its center at the origin.
µ0 nIR2
µ0 nI πR2 = 2πsAφ ⇒ Ap hi = (3.4)
2s

2
We want the field at z = d. The magnetic field from a current The field of a dipole is
distribution is µ0 m
Bdip = (2 cos θr̂ + sin θθ̂)
dl0 × (r − r0 ) 4π r3
Z
µ0 I
B=
4π |r − r0 |3/2 At r = d, θ = 0, and m = w2 I,
Consider the contribution from the sides that extends from
µ0 w 2 I
x = −w/2 to x = w/2 at y = w/2. Bdip = r̂
2π d3
dl0 = x̂dx

r = dẑ
w 5. Magnetized cylinder
r0 = xx̂ + ŷ
2
Griffiths 6.12. An infinitely long cylinder, of radius R, carries
Then
w a “frozen-in” magnetization, parallel to the axis,
dl0 × (r − r0 ) = (− ẑ + dŷ)
2
Next M = ksẑ,
0 3 2 2 2 3/2
|r − r | = (x + (w/2) + d )
where k is a constant and s is the distance from the axis; there
Putting the pieces together we have the contribution to the is no free current anywhere. Find the magnetic field inside and
z-component of the field from one size is
outside the cylinder by two different methods:
Z −w/2 −w
µ0 2 dx 1. As in Section 6.2, locate all of the bound currents, and
Bz (z = d) = I
4π w/2 (x + (w/2)2 + d2 )3/2
2
calculate the field they produce.
µ0 I w2 2. Use Ampere’s law (in the form of Eq. 6.20) to find H,
=
8π ((w/2)2 + d2 )(w2 /2 + d2 )1/2 and then get B from Eq. 6.18. (Notice that the second
Each side will contribute the same. The total field in the method is much faster, and avoids any explicit reference
z-direction will be to the bound currents.)
µ0 I w2
Bz = SOLUTION:
2π ((w/2)2 + d2 )(w2 /2 + d2 )1/2
a) There can be bound current density in the cylinder and a
The net y-component of the field will be zero by symmetry. bound surface current at the boundary of the cylinder s = R.
In the limit z  w
Bz =
µ0 I w 2 J~b = ∇ ~ = 1 ∂Mz ŝ − ∂Mz φ̂ = −k φ̂
~ ×M (5.1)
2π d3 s ∂φ ∂s

3
~b = M
K ~ × n̂ = kR ẑ × ŝ = kR φ̂ (5.2)
We can think of cylindrical shells of thickness dz inside the
cylinder. If we divide up the cylinder like this, then we have
a set of concentric solenoids with surface current J~b ds plus
a solenoid with surface current K~ b at the outside. The mag-
netic field at any point will have contributions from each of
the solenoids that surrounds it. Just as a reminder, a solenoid
with surface current K φ̂ has magnetic field B = µ0 K ẑ in-
side. The magnetic field outside the cylinder is 0. Inside the
cylinder we have:
Z R Z R
~ = µ0 Kb ẑ + µ0 ẑ
B Jb ds = µ0 kR ẑ − µ0 ẑ k ds (5.3)
s s

~ = (µ0 kR − µ0 kR + µ0 ks) ẑ = µ0 ks ẑ
B (5.4)

~ · d~` = If enc . Since


H
b) Here we just use Ampère’s law. H
~ = 0 everywhere. Now
there is no free current anywhere, H
we use the relation:

~ = 1B
H ~ −M
~ (5.5)
µ0

~ = µ0 M
B ~ = µ0 ks ẑ (5.6)

You might also like